LSAT

This topic has expert replies
Master | Next Rank: 500 Posts
Posts: 338
Joined: Fri Apr 17, 2009 1:49 am
Thanked: 9 times
Followed by:3 members

LSAT

by kaulnikhil » Thu Sep 10, 2009 5:24 am
The workers at Bell Manufacturing will shortly go on strike unless the management increases their wages. As Bell's president is well aware, however, in order to increase the worker's wages, Bell would have to sell off some of its subsidiaries. So, some of Bell's subsidiaries will be sold.
The conclusion above is properly drawn if which one of the following is assumed?
(A) Bell Manufacturing will begin to suffer increased losses.
(B) Bell's management will refuse to increase its worker's wages.
(C) The workers at Bell Manufacturing will not be going on strike.
(D) Bell's president has the authority to offer the workers their desired wage increase.
(E) Bell's workers will not accept a package of improved benefits in place of their desired wage increase.

Senior | Next Rank: 100 Posts
Posts: 63
Joined: Mon Jul 20, 2009 11:48 am
GMAT Score:730

by rainmaker » Thu Sep 10, 2009 6:34 am
I'm confused between D and E. Whats the OA?

Master | Next Rank: 500 Posts
Posts: 159
Joined: Thu Aug 27, 2009 10:30 am
Thanked: 19 times

by bharathh » Thu Sep 10, 2009 7:04 am
I'll go with D. It says that the subsidiaries were sold... so a wage increase was inevitable. Only D makes that clear. Benefits would cost more as well but there is nothing that alludes to benefits being offered in the passage.

Master | Next Rank: 500 Posts
Posts: 189
Joined: Thu Apr 03, 2008 2:03 pm
Location: USA
Thanked: 21 times

by rohan_vus » Thu Sep 10, 2009 7:18 am
IMO C . Denying C makes the argument fall apart so hence right in my opinion . Looking foward to OA

Legendary Member
Posts: 876
Joined: Thu Apr 10, 2008 8:14 am
Thanked: 13 times

by ketkoag » Thu Sep 10, 2009 9:00 am
C it is.. C is definitely the conclusion coz it directly fills the unstated premise that is necessary for this argument to be valid..

Master | Next Rank: 500 Posts
Posts: 338
Joined: Fri Apr 17, 2009 1:49 am
Thanked: 9 times
Followed by:3 members

by kaulnikhil » Thu Sep 10, 2009 9:21 am
OA C
Can u plz provide explanation

Master | Next Rank: 500 Posts
Posts: 160
Joined: Sun May 03, 2009 1:17 am
Location: Rourkela/Hyderabad
Thanked: 4 times
Followed by:1 members

by sanp_l » Thu Sep 10, 2009 10:07 am
The question above asks to identify the assumption made for concluding that the subsidiaries will be sold. That case is true when increase in wages is certain. That implies that there is no scope that the workers are going on strike. Hence option C stands correct.

Nice Question.
Sandy

Legendary Member
Posts: 876
Joined: Thu Apr 10, 2008 8:14 am
Thanked: 13 times

Re: LSAT

by ketkoag » Thu Sep 10, 2009 10:09 am
kaulnikhil wrote:The workers at Bell Manufacturing will shortly go on strike unless the management increases their wages. As Bell's president is well aware, however, in order to increase the worker's wages, Bell would have to sell off some of its subsidiaries. Since, The workers at Bell Manufacturing will not be going on strike, some of Bell's subsidiaries will be sold.



(A) Bell Manufacturing will begin to suffer increased losses.
(B) Bell's management will refuse to increase its worker's wages.
(C) The workers at Bell Manufacturing will not be going on strike.
(D) Bell's president has the authority to offer the workers their desired wage increase.
(E) Bell's workers will not accept a package of improved benefits in place of their desired wage increase.
see the bold faced part above..
try to put all the answer choices in that place and check which one makes sense and thus makes the argument more credible..
HTH

Junior | Next Rank: 30 Posts
Posts: 19
Joined: Sat Jan 10, 2009 10:58 pm

Re: LSAT

by [email protected] » Thu Sep 10, 2009 12:17 pm
ketkoag wrote:
kaulnikhil wrote:The workers at Bell Manufacturing will shortly go on strike unless the management increases their wages. As Bell's president is well aware, however, in order to increase the worker's wages, Bell would have to sell off some of its subsidiaries. Since, The workers at Bell Manufacturing will not be going on strike, some of Bell's subsidiaries will be sold.
HTH
What if the management does not inc wages, so workers go on strike... seeing this management bows to workers demands and inc workers' wages.. :wink:

Master | Next Rank: 500 Posts
Posts: 371
Joined: Tue Apr 29, 2008 10:16 am
Thanked: 6 times
Followed by:1 members

by vaivish » Sun Oct 10, 2010 10:04 am
Just an another thought on option D.

If the president does not have authority, how can the wages be increased. C is already mentioned in the passage as indicated by someone. As per the definition, Assumption should be unstated. I htink its should be D. Any thoughts on above. /

User avatar
GMAT Instructor
Posts: 2193
Joined: Mon Feb 22, 2010 6:30 pm
Location: Vermont and Boston, MA
Thanked: 1186 times
Followed by:512 members
GMAT Score:770

by David@VeritasPrep » Sun Oct 10, 2010 12:29 pm
This particular type of LSAT question is not found on the GMAT. If you do use LSAT questions to study you should keep in mind that there are three types of strengthen questions on the LSAT and only two on the GMAT. This is how you tell the difference:

If you see the word "most" in the question stem and you are sure that you have a strengthen question, then this is a regular strengthen question and you can probably approach it like a GMAT question.

If you find that argument makes an assumption or if the argument depends on something then you likely have an assumption question, also found on the GMAT. Just remember one thing -- if you can replace the word "assumed" with the word "true" then you can ignore the assume and it is not an assumption question.

That is the case here. The question stem says, "The conclusion above is properly drawn if which one of the following is assumed?" You can replace the "assumed" with "true" so you can ignore it. That means that this is not an assumption question!

That makes this question fall into the third category, the one that is not on the GMAT. The correct answer to this type of question will not only strengthen the conclusion but will actually guarantee it. That is what "properly drawn" means. You can usually treat these questions as if they were strengthen questions but there is a big difference and that is that it is possible that more than one answer choice will strengthen the question, while only one will technically have the conclusion "properly drawn" or "must be true."

For this question, You are looking for the statement that guarantees that the some of the subsidiaries will be sold. How can this be guaranteed? Well we have some steps in the process and we need one more..Here is what we have; We know that the only way the workers do not go on strike is if the wages are increased. We know that only way Bell can increase the wages is to sell off some subsidiaries. So we are nearly there to our conclusion. Answer choice C supplies the missing link (as ketkoag pointed out) If we know that the workers did not go on strike and the only way this is prevented is with a wage increase and that can only happen if subsidiaries are sold, well we are there!

Please be careful with a question like this. Choice C is not the correct choice for an assumption question as there could be other ways to get to the subsidiaries being sold. And this question relies on formal logic more than we would ideally want because this is not tested on the GMAT.

I hope that helps you to use LSAT questions if you choose to...
Veritas Prep | GMAT Instructor

Veritas Prep Reviews
Save $100 off any live Veritas Prep GMAT Course

Legendary Member
Posts: 2330
Joined: Fri Jan 15, 2010 5:14 am
Thanked: 56 times
Followed by:26 members

by mundasingh123 » Mon Oct 11, 2010 2:33 am
Hi david,
1)can you please explain why D is eliminated.The president is aware of the fact that selling the subsidiaries is the only way to prevent workers from going on strike.What is the purpose of including "the president " in the premise.
So, if the president has the authority to take the decision to sell the subsidiaries, he will sell the subsidiaries.

2)Also, as you said that if the word assumed can be replaced by the word "true" then its not an assumption question.
I thought anything to do with assume is a question marker.Do you have other exceptions where a minor change in the question stem makes it different from an assumption question.

User avatar
GMAT Instructor
Posts: 2193
Joined: Mon Feb 22, 2010 6:30 pm
Location: Vermont and Boston, MA
Thanked: 1186 times
Followed by:512 members
GMAT Score:770

by David@VeritasPrep » Mon Oct 11, 2010 4:55 am
On the GMAT (and on official GMAT questions) anytime you see any variation on the word "assume" it is definitely an assumption question.

One of the pitfalls of looking at LSAT questions is that they may create some confusion. In the case of LSAT questions only, not GMAT questions, if you can replace the word "assumes" with the word "true" then you can ignore the assumes.

As to why the president is mentioned in the stimulus, this is a common tactic on the LSAT that might occur on really difficult GMAT problems -- there was no reason to mention the president, and if you look at the reasoning, the president of the company in no way figures in, that was just mentioned to set up answer choice D as a distraction.

vaivish treated this as an assumption question, he said, "If the president does not have authority, how can the wages be increased?" This is not the way to approach this type of question. Treat this more as a strengthen and not an assumption. Adding the fact that the president does have the authority does not mean that she will use it. So this is not a good strengthen answer.

As a side note, I do not actually believe D is the right answer to an assumption either. What if the President does not have the authority? What if she has to get the permission of the Board of Directors? Does that mean that the subsidiaries cannot be sold? A better assumption answer could be "the company owns subsidiaries" or "it is possible to sell subsidiaries without selling the entire company." These statements, if negated, would directly attack the conclusion.

Let me just reiterate that this type of question is not on the GMAT. If this one confuses you then ignore it you will not see it on test day!
Veritas Prep | GMAT Instructor

Veritas Prep Reviews
Save $100 off any live Veritas Prep GMAT Course